LSAT and Law School Admissions Forum

Get expert LSAT preparation and law school admissions advice from PowerScore Test Preparation.

 rleung
  • Posts: 3
  • Joined: Aug 26, 2016
|
#28526
Hi,
I am having a little trouble understanding the logic behind 23. I chose C because it made sense that the rare motorcycle parts would be similar to rare tulip bulbs, and then the person in answer C would be forced to sell it at a lower price after parts become more available and widespread. I see that D is the right answer, but am a little confused because it doesn't say that the publisher is forced to sell it at a more affordable price due to outside circumstances.
Thanks!
 Adam Tyson
PowerScore Staff
  • PowerScore Staff
  • Posts: 5153
  • Joined: Apr 14, 2011
|
#28545
The important thing to keep in mind in this question is "who" - whose view are we examining here? Tracking the viewpoints is crucial in this passage.

This question asks us about Garber's view, not Mackay's. Garber is the one who tells us that the Dutch tulip market did NOT represent an irrational bubble, but rather made sense and reflected the fundamentals of a normal market. The original high price is rational based on the popularity of the original bulb, and the owner of that bulb gets to recoup his investment over time by selling reproductions of the original (descendants of the original bulb) at much lower prices, making up in volume what he loses in price per copy.

In Garber's analysis, nobody is forced to do anything, but rather they plan and choose to pay the early high price in order to get the long-term gain of selling many copies for less per copy. That's what is happening in answer D - the publisher who owns the right to the novel's manuscript doesn't mind paying a lot to get it, because he alone can collect the long-term sales revenues from the copies.

Pay careful attention to the viewpoints here - Mackay might have said that the tulip market was a bubble, and answer C might look like his analysis, but since the question asked about Garber's account you have to focus on his analysis that the market was rational and fundamental, not a bubble at all.
 LAM
  • Posts: 41
  • Joined: Dec 15, 2016
|
#33695
Wow. That was an excellent explanation. I too got this wrong, and I too chose Answer C. But this makes it crystal clear why I was wrong - and in fact was agreeing with Mackay without even knowing it. C is essentially describing a bubble. Thanks for the clarification.
User avatar
 ashpine17
  • Posts: 321
  • Joined: Apr 06, 2021
|
#99490
it seems that "cheap substitute parts" was the problem with C since the passage talked about reproductions of the original tulip bulb instead of a substitute? is that the only issue with this choice?
User avatar
 ashpine17
  • Posts: 321
  • Joined: Apr 06, 2021
|
#99492
i'm looking for more reasons that C is wrong...is the reason why the person with the parts decided to sell them match the original reason the Original tulip bulb owner decided to sell the tulip bulbs? my impression was that the tulip bulb owner was trying to make back profits
 Robert Carroll
PowerScore Staff
  • PowerScore Staff
  • Posts: 1787
  • Joined: Dec 06, 2013
|
#100482
ashpine,

That's a serious issue with answer choice (C). As pointed out earlier in this thread, answer choice (C) seems to be describing a speculative bubble situation - someone buys things at a high price and is forced to sell them later at a low price because they've lost value. The person doing that isn't investing well. But Garber viewed the tulip craze as rational - not a speculative bubble at all. So answer choice (C) is a complete non-starter.

In the passage, the tulip buyer is trying to profit. Garber indicates that they may have been able to profit even if they bought an original bulb for a lot of money, yet descendants of that bulb ended up being cheap. It's because the single expensive bulb had bunches of cheap offspring. No one's necessarily losing money in that situation - buying one bulb for a million dollars, then selling a million of its offspring for two dollars each can represent a profit.

Answer choice (C) doesn't have any even apparent profit in it. It looks just like a loss.

Robert Carroll

Get the most out of your LSAT Prep Plus subscription.

Analyze and track your performance with our Testing and Analytics Package.